[obm-l] Re: [obm-l] Re: [obm-l] Re: [obm-l] Congruência (?)

2019-10-09 Por tôpico Pedro José
Boa noite!
Faltara também a explicação.
Seja a = r  mod 10 então a^n=(r)^n  mod 100  se n é múltiplo de 10.
Mas é só usar o binômio de Newton, para (10q+r)^n  só sobra o último termo.

Saudações.

Em qua, 9 de out de 2019 às 11:09, Pedro José 
escreveu:

> Bom dia!
>
> Achei um outro modo de resolver, só que ao retornar me apercebi de que
> "engolira a classe 6', ao invés de ir na PA(2,4,6,8) segui pela PG (2,4,8)
>
> Faltou então para o algarismo 6.
>
> 6^20=2^20.3^20 e ord1003=20então 2^20= 1 mod 100 então 6=^20=2^20 mod
> 100
> Se 3^n= 1 mod100 então 3^n= 1 mod10
> ord103=4
> (3)^n=1 mod100 então né múltiplo de 4. Então n=4k par k>1 inteiro.
> (3)^n=(81)^k=(10*8+1)^k
> Pelo binômio de Newton, só sobram os dois últimos termos. Os demais terão
> 10^m com m>2 que côngruo de 0  mod100
> k.10*8 +1, e portanto o menor k que satisfaz é k=5. Então ord1003=20
>
> Com isso completa o que faltara da resolução anterior.
>
> 2^10=1024=24 mod100
> 2^20=24^2=76 mod100
> 4^20=(2^20)^2=76^2=(-24)^2=576=76 mod100
> 8^20=2^20.4^20=76^2=24 mod100
> 6^20=3^20.2^20=2^20 pois ord1003=20
>
> Essa última ficou melhor.
>
> Saudações,
> PJMS
>
>
> Em sáb, 5 de out de 2019 às 08:58, Pedro José 
> escreveu:
>
>> Bom dia!
>> Esdras, tem como postar a resposta.
>> Não consigo ver a^p=a modp, para p primo se encaixando no problema, pois
>> 10 não é primo.
>>
>> Grato!
>>
>> Saudações,
>> PJMS
>>
>>
>> Em sex, 4 de out de 2019 às 20:20, Esdras Muniz <
>> esdrasmunizm...@gmail.com> escreveu:
>>
>>> Dá pra fazer tb usando o pequeno teorema de Fermat.
>>>
>>>
>>> 
>>>  Livre
>>> de vírus. www.avast.com
>>> .
>>>
>>> <#m_3285326544539962876_m_-140568092169550719_m_1063528150960112747_m_-1601668305501320773_m_-5542290881960747167_m_-611650024147786599_DAB4FAD8-2DD7-40BB-A1B8-4E2AA1F9FDF2>
>>>
>>> Em sex, 4 de out de 2019 às 17:36, Pedro José 
>>> escreveu:
>>>
 Boa tarde!
 Com minhas escusas retificação da solução.
 n<>o mod10 e não: "n<> 0 mod100"
 (100,4) <>1 e não: "(100,4) =1"
 b^x não se repete e não: "b^x não se repetem"
 Sds,
 PJMS.


 Em sex, 4 de out de 2019 às 17:16, Pedro José 
 escreveu:

> Boa tarde!
> Se 10 não divide n então n<>0 mod100; pois nesse caso daria "00".
> Então os números são 2,4 ou 8 côngruos mod10.
> 2^20=4^10
> 8^20 = 4^40
> 4^1= 4 mod10
> 4^2=6 mod10
> 4^3= 4 mod10
> Logo temos que 4^(2m+1) = 4 mod 10 (i)
> Se
> a=4 mod 100 ==> a=4 mod 10 (ii)
>
> Então vamos procurar o período de a^n mod100,  Não existe a que
> satisfaça a^m= 1 mod100, com m<>0, pois (100,4)=1
> Vamos tentar verificar se há repetição do 4.
> De (i) e (ii) , temos que: 4^(2m+1) = 4 mod 100
> m=1 ==> 4^3 = 64 mod 104, não serve
> m=2 ==> 4^5= (4^(3*2))*8 = 28*8= 224=24 mod 100, não serve
> m=3 ==> 4^7= 24*16=384=84 mod 100
> m=4 ==> 4^9= (2*84)*8=68*8= 544=44 mod100
> m=5 ==> 4^11=44*16= 704= 4 mod 100
> Portanto o período de 4^a mod100 é 1gual a 10, ou seja, 4^a=4(10x+a)
> mod100. com x,a não nulos (Cuidado, que para alguns casos em que (b,m)<>1,
> b^x não se repetem para x < xo,e.g., 2^a= 2 mod 100, só é atendido para
> a=1, aí tem de sair no braço para ver qual que se repete e pode-se gastar
> mais tempo. Por sorte o quatro repetiu. Mas o enunciado dava a dica de que
> repetiria, pois, 4^20=4^10 mod 100 para que o problema tenha uma solução
> única.
> 4^20 = 4^10= 4^9*4=44*4=176=76 mod100
> 8^20=4^40=4^10=76 mod100
> 2^20=4^10=76 mod 100.
>
> Portanto o algarismo da dezena é 7 e das unidades 6.
>
> Saudações,
> PJMS
>
>
>
>
> Em qui, 3 de out de 2019 às 17:51, marcone augusto araújo borges <
> marconeborge...@hotmail.com> escreveu:
>
>> Se n é um número natural par não divisível por 10, quais são os dois
>> últimos algarismos de n^20?
>> --
>> Esta mensagem foi verificada pelo sistema de antivírus e
>> acredita-se estar livre de perigo.
>>
>
 --
 Esta mensagem foi verificada pelo sistema de antivírus e
 acredita-se estar livre de perigo.
>>>
>>>
>>>
>>> --
>>> Esdras Muniz Mota
>>> Mestrando em Matemática
>>> Universidade Federal do Ceará
>>>
>>>
>>>
>>> --
>>> Esta mensagem foi verificada pelo sistema de antivírus e
>>> acredita-se estar livre de perigo.
>>
>>

-- 
Esta mensagem foi verificada pelo sistema de antiv�rus e
 acredita-se estar livre de perigo.



[obm-l] Re: [obm-l] Re: [obm-l] Re: [obm-l] Congruência (?)

2019-10-09 Por tôpico Pedro José
Bom dia!

Achei um outro modo de resolver, só que ao retornar me apercebi de que
"engolira a classe 6', ao invés de ir na PA(2,4,6,8) segui pela PG (2,4,8)

Faltou então para o algarismo 6.

6^20=2^20.3^20 e ord1003=20então 2^20= 1 mod 100 então 6=^20=2^20 mod100
Se 3^n= 1 mod100 então 3^n= 1 mod10
ord103=4
(3)^n=1 mod100 então né múltiplo de 4. Então n=4k par k>1 inteiro.
(3)^n=(81)^k=(10*8+1)^k
Pelo binômio de Newton, só sobram os dois últimos termos. Os demais terão
10^m com m>2 que côngruo de 0  mod100
k.10*8 +1, e portanto o menor k que satisfaz é k=5. Então ord1003=20

Com isso completa o que faltara da resolução anterior.

2^10=1024=24 mod100
2^20=24^2=76 mod100
4^20=(2^20)^2=76^2=(-24)^2=576=76 mod100
8^20=2^20.4^20=76^2=24 mod100
6^20=3^20.2^20=2^20 pois ord1003=20

Essa última ficou melhor.

Saudações,
PJMS


Em sáb, 5 de out de 2019 às 08:58, Pedro José 
escreveu:

> Bom dia!
> Esdras, tem como postar a resposta.
> Não consigo ver a^p=a modp, para p primo se encaixando no problema, pois
> 10 não é primo.
>
> Grato!
>
> Saudações,
> PJMS
>
>
> Em sex, 4 de out de 2019 às 20:20, Esdras Muniz 
> escreveu:
>
>> Dá pra fazer tb usando o pequeno teorema de Fermat.
>>
>>
>> 
>>  Livre
>> de vírus. www.avast.com
>> .
>>
>> <#m_-140568092169550719_m_1063528150960112747_m_-1601668305501320773_m_-5542290881960747167_m_-611650024147786599_DAB4FAD8-2DD7-40BB-A1B8-4E2AA1F9FDF2>
>>
>> Em sex, 4 de out de 2019 às 17:36, Pedro José 
>> escreveu:
>>
>>> Boa tarde!
>>> Com minhas escusas retificação da solução.
>>> n<>o mod10 e não: "n<> 0 mod100"
>>> (100,4) <>1 e não: "(100,4) =1"
>>> b^x não se repete e não: "b^x não se repetem"
>>> Sds,
>>> PJMS.
>>>
>>>
>>> Em sex, 4 de out de 2019 às 17:16, Pedro José 
>>> escreveu:
>>>
 Boa tarde!
 Se 10 não divide n então n<>0 mod100; pois nesse caso daria "00".
 Então os números são 2,4 ou 8 côngruos mod10.
 2^20=4^10
 8^20 = 4^40
 4^1= 4 mod10
 4^2=6 mod10
 4^3= 4 mod10
 Logo temos que 4^(2m+1) = 4 mod 10 (i)
 Se
 a=4 mod 100 ==> a=4 mod 10 (ii)

 Então vamos procurar o período de a^n mod100,  Não existe a que
 satisfaça a^m= 1 mod100, com m<>0, pois (100,4)=1
 Vamos tentar verificar se há repetição do 4.
 De (i) e (ii) , temos que: 4^(2m+1) = 4 mod 100
 m=1 ==> 4^3 = 64 mod 104, não serve
 m=2 ==> 4^5= (4^(3*2))*8 = 28*8= 224=24 mod 100, não serve
 m=3 ==> 4^7= 24*16=384=84 mod 100
 m=4 ==> 4^9= (2*84)*8=68*8= 544=44 mod100
 m=5 ==> 4^11=44*16= 704= 4 mod 100
 Portanto o período de 4^a mod100 é 1gual a 10, ou seja, 4^a=4(10x+a)
 mod100. com x,a não nulos (Cuidado, que para alguns casos em que (b,m)<>1,
 b^x não se repetem para x < xo,e.g., 2^a= 2 mod 100, só é atendido para
 a=1, aí tem de sair no braço para ver qual que se repete e pode-se gastar
 mais tempo. Por sorte o quatro repetiu. Mas o enunciado dava a dica de que
 repetiria, pois, 4^20=4^10 mod 100 para que o problema tenha uma solução
 única.
 4^20 = 4^10= 4^9*4=44*4=176=76 mod100
 8^20=4^40=4^10=76 mod100
 2^20=4^10=76 mod 100.

 Portanto o algarismo da dezena é 7 e das unidades 6.

 Saudações,
 PJMS




 Em qui, 3 de out de 2019 às 17:51, marcone augusto araújo borges <
 marconeborge...@hotmail.com> escreveu:

> Se n é um número natural par não divisível por 10, quais são os dois
> últimos algarismos de n^20?
> --
> Esta mensagem foi verificada pelo sistema de antivírus e
> acredita-se estar livre de perigo.
>

>>> --
>>> Esta mensagem foi verificada pelo sistema de antivírus e
>>> acredita-se estar livre de perigo.
>>
>>
>>
>> --
>> Esdras Muniz Mota
>> Mestrando em Matemática
>> Universidade Federal do Ceará
>>
>>
>>
>> --
>> Esta mensagem foi verificada pelo sistema de antivírus e
>> acredita-se estar livre de perigo.
>
>

-- 
Esta mensagem foi verificada pelo sistema de antiv�rus e
 acredita-se estar livre de perigo.



[obm-l] Re: [obm-l] Re: [obm-l] Re: [obm-l] Congruência (?)

2019-10-05 Por tôpico Pedro José
Bom dia!
Esdras, tem como postar a resposta.
Não consigo ver a^p=a modp, para p primo se encaixando no problema, pois 10
não é primo.

Grato!

Saudações,
PJMS


Em sex, 4 de out de 2019 às 20:20, Esdras Muniz 
escreveu:

> Dá pra fazer tb usando o pequeno teorema de Fermat.
>
>
> 
>  Livre
> de vírus. www.avast.com
> .
>
> <#m_-5542290881960747167_m_-611650024147786599_DAB4FAD8-2DD7-40BB-A1B8-4E2AA1F9FDF2>
>
> Em sex, 4 de out de 2019 às 17:36, Pedro José 
> escreveu:
>
>> Boa tarde!
>> Com minhas escusas retificação da solução.
>> n<>o mod10 e não: "n<> 0 mod100"
>> (100,4) <>1 e não: "(100,4) =1"
>> b^x não se repete e não: "b^x não se repetem"
>> Sds,
>> PJMS.
>>
>>
>> Em sex, 4 de out de 2019 às 17:16, Pedro José 
>> escreveu:
>>
>>> Boa tarde!
>>> Se 10 não divide n então n<>0 mod100; pois nesse caso daria "00".
>>> Então os números são 2,4 ou 8 côngruos mod10.
>>> 2^20=4^10
>>> 8^20 = 4^40
>>> 4^1= 4 mod10
>>> 4^2=6 mod10
>>> 4^3= 4 mod10
>>> Logo temos que 4^(2m+1) = 4 mod 10 (i)
>>> Se
>>> a=4 mod 100 ==> a=4 mod 10 (ii)
>>>
>>> Então vamos procurar o período de a^n mod100,  Não existe a que
>>> satisfaça a^m= 1 mod100, com m<>0, pois (100,4)=1
>>> Vamos tentar verificar se há repetição do 4.
>>> De (i) e (ii) , temos que: 4^(2m+1) = 4 mod 100
>>> m=1 ==> 4^3 = 64 mod 104, não serve
>>> m=2 ==> 4^5= (4^(3*2))*8 = 28*8= 224=24 mod 100, não serve
>>> m=3 ==> 4^7= 24*16=384=84 mod 100
>>> m=4 ==> 4^9= (2*84)*8=68*8= 544=44 mod100
>>> m=5 ==> 4^11=44*16= 704= 4 mod 100
>>> Portanto o período de 4^a mod100 é 1gual a 10, ou seja, 4^a=4(10x+a)
>>> mod100. com x,a não nulos (Cuidado, que para alguns casos em que (b,m)<>1,
>>> b^x não se repetem para x < xo,e.g., 2^a= 2 mod 100, só é atendido para
>>> a=1, aí tem de sair no braço para ver qual que se repete e pode-se gastar
>>> mais tempo. Por sorte o quatro repetiu. Mas o enunciado dava a dica de que
>>> repetiria, pois, 4^20=4^10 mod 100 para que o problema tenha uma solução
>>> única.
>>> 4^20 = 4^10= 4^9*4=44*4=176=76 mod100
>>> 8^20=4^40=4^10=76 mod100
>>> 2^20=4^10=76 mod 100.
>>>
>>> Portanto o algarismo da dezena é 7 e das unidades 6.
>>>
>>> Saudações,
>>> PJMS
>>>
>>>
>>>
>>>
>>> Em qui, 3 de out de 2019 às 17:51, marcone augusto araújo borges <
>>> marconeborge...@hotmail.com> escreveu:
>>>
 Se n é um número natural par não divisível por 10, quais são os dois
 últimos algarismos de n^20?
 --
 Esta mensagem foi verificada pelo sistema de antivírus e
 acredita-se estar livre de perigo.

>>>
>> --
>> Esta mensagem foi verificada pelo sistema de antivírus e
>> acredita-se estar livre de perigo.
>
>
>
> --
> Esdras Muniz Mota
> Mestrando em Matemática
> Universidade Federal do Ceará
>
>
>
> --
> Esta mensagem foi verificada pelo sistema de antivírus e
> acredita-se estar livre de perigo.

-- 
Esta mensagem foi verificada pelo sistema de antiv�rus e
 acredita-se estar livre de perigo.



[obm-l] Re: [obm-l] Re: [obm-l] Re: [obm-l] Congruência módulo m

2014-05-02 Por tôpico Pedro José
Boa tarde!

Ruy,

Observe que são onze classe de congruência módulo 11:

Não tenho como colocar a barra acima dos números, mas enxergue a barra.

0 = {...-33, -22, -11, 0, 11, 22, 33...}
1 = {-32, -21, -10, 1, 12, 23, 34}

E assim sucessivamente até 10 = {...-23, -12, -1, 10, 21, 32...}


É fácil provar que as classes módulo m preservam a adição, basta usar
divisão de Euclides e fechamento da adição(por tabela  fechamento da
multiplicação) em Z.

Se preservam a adição preservam a multiplicação e a potenciação.

Portanto qualquer elemento de uma classe de congruência elevado a um dado
inteiro terá a mesma congruência módulo p.

Razão pela qual o colega informou que bastam serem verificados 11 valores
para congruência módulo 11.

Saudações,
PJMS





Em 2 de maio de 2014 08:15, Pacini Bores pacini.bo...@globo.com escreveu:

 Observe que são apenas 11 valores para  a devida verificação, portanto sem
 grandes trabalhos, ok ?

 Pacini


 Em 2 de maio de 2014 01:43, ruymat...@ig.com.br escreveu:

  Módulo 11.




 Em 02/05/2014 00:49, Cassio Anderson Feitosa escreveu:

  Em qual módulo?

 Em 2 de maio de 2014 00:42, ruymat...@ig.com.br escreveu:

  É fácil ver que para todo inteiro x, x^5 é côngruo a -1, 0 e 1 apenas.
 Mas como prova-lo para todos sem ter que testar um a um dos possíveis
 valores de x ( x=1,2,3,4,5,6,...)? Abraços e agradecimentos antecipados a
 quem responder .

R.O.




 --
 Esta mensagem foi verificada pelo sistema de antivírus e
 acredita-se estar livre de perigo.




 --
 Cássio Anderson
 Graduando em Matemática - UFPB

 --
 Esta mensagem foi verificada pelo sistema de antiv?rus e
 acredita-se estar livre de perigo.


 --
 Esta mensagem foi verificada pelo sistema de antivírus e
 acredita-se estar livre de perigo.



 --
 Esta mensagem foi verificada pelo sistema de antivírus e
 acredita-se estar livre de perigo.


-- 
Esta mensagem foi verificada pelo sistema de antiv�rus e
 acredita-se estar livre de perigo.



[obm-l] RE: [obm-l] Re: [obm-l] RE: [obm-l] Congruência

2011-11-28 Por tôpico João Maldonado


Como você  disse,  13^4 = 1  mod(10), analisando 13, ou 3, mod 10,  concluimos 
que 13^n = 1 mod(10) se n = 4k,  ou seja, se n é múltiplo de 4.   Mas 9^9 não é 
múltiplo de 4
Para  ficar  mais claro
13^0 = 113^0 = 113^0 = 113^0 = 113^0 = 113^0 = 113^0 = 113^0 = 113^0 = 113^0 = 
113^0 = 113^0 = 113^0 = 1
Date: Mon, 28 Nov 2011 07:27:19 -0200
Subject: [obm-l] Re: [obm-l] RE: [obm-l] Congruência
From: klebe...@gmail.com
To: obm-l@mat.puc-rio.br

Bom dia, pensei assim:

13 = 3 mod(10)
13^2 = -1 mod(10)
13^4 = -1^2 mod(10)
13^4 = 1 mod(10)
(13)^9^9 = (1)^9^9 mod(10)
(13)^9^9 = 1 mod(10)

Ou seja, Resto igual a 1. Que será o último algarismo.

Será que tá errado?

Abraços, Kleber.

2011/11/28 João Maldonado joao_maldona...@hotmail.com







Se for  13^(9^9) mod(10) = 3^(9^9) mod(10)
Vamos analisar 3^x  mod 103^0 = 1  (4k)3^1 = 3  (4k+1)3^2 = 9  (4k+2)3^3 = 7  
(4k+3)

9^9  mod(4) = 1^9 mod(4) = 1
Logo  9^9  = 4k+1 e  3^(4k+1 = 3 mod(10)
Resposta: 3

Date: Sun, 27 Nov 2011 21:13:15 -0200
Subject: [obm-l] Congruência

From: klebe...@gmail.com
To: obm-l@mat.puc-rio.br

Olá amigos,

O exercício é simples, mas não estou conseguindo visualizar essa solução.


Achar o último algarismo de (13)^9^9 (13 elevado a 9^9).

Desde de já agradeço a ajuda.

Abraços, 

-- 
Kleber (Ps. fiz por congruência módulo 10, mas não cheguei a conclusão)
  


-- 
Kleber B. Bastos
  

[obm-l] RE: [obm-l] Re: [obm-l] RE: [obm-l] Congruência

2011-11-28 Por tôpico João Maldonado

Desculpe pelo outro email, saiu errado
Como você  disse,  13^4 = 1  mod(10), analisando 13, ou 3, mod 10,  concluimos 
que 13^n = 1 mod(10) se n = 4k,  ou seja, se n é múltiplo de 4.   Mas 9^9 não é 
múltiplo de 4Portanto é impossível que 13^(9^9) = 1 mod(10)
Para  ficar  mais claro
13^0 = 1  (4k)13^1 = 3  (4k+1)13^2 = 9  (4k+2)13^3 = 7  (4k+3)13^4 = 1  
(4k)13^5 = 3  (4k+1)13^6 = 9  (4k+2)13^7 = 7  (4k+3)13^8 = 1  (4k)13^9 = 3  
(4k+1)13^10 = 9  (4k+2)13^11 = 7  (4k+3)13^12 = 1  (4k)13^13 = 3  (4k+1)...
O 1, 3, 9, 7  vai se repetindo
Não entendi essa passagem que você fez
13^4 = 1 mod(10)
(13)^9^9 = (1)^9^9 mod(10)

Sim, 13^4 = 1 mod(10)Mas 13 não
[]'sJoao
Date: Mon, 28 Nov 2011 07:27:19 -0200
Subject: [obm-l] Re: [obm-l] RE: [obm-l] Congruência
From: klebe...@gmail.com
To: obm-l@mat.puc-rio.br

Bom dia, pensei assim:

13 = 3 mod(10)
13^2 = -1 mod(10)
13^4 = -1^2 mod(10)
13^4 = 1 mod(10)
(13)^9^9 = (1)^9^9 mod(10)
(13)^9^9 = 1 mod(10)

Ou seja, Resto igual a 1. Que será o último algarismo.

Será que tá errado?

Abraços, Kleber.

2011/11/28 João Maldonado joao_maldona...@hotmail.com







Se for  13^(9^9) mod(10) = 3^(9^9) mod(10)
Vamos analisar 3^x  mod 103^0 = 1  (4k)3^1 = 3  (4k+1)3^2 = 9  (4k+2)3^3 = 7  
(4k+3)

9^9  mod(4) = 1^9 mod(4) = 1
Logo  9^9  = 4k+1 e  3^(4k+1 = 3 mod(10)
Resposta: 3

Date: Sun, 27 Nov 2011 21:13:15 -0200
Subject: [obm-l] Congruência

From: klebe...@gmail.com
To: obm-l@mat.puc-rio.br

Olá amigos,

O exercício é simples, mas não estou conseguindo visualizar essa solução.


Achar o último algarismo de (13)^9^9 (13 elevado a 9^9).

Desde de já agradeço a ajuda.

Abraços, 

-- 
Kleber (Ps. fiz por congruência módulo 10, mas não cheguei a conclusão)
  


-- 
Kleber B. Bastos
  

[obm-l] Re: [obm-l] RE: [obm-l] Re: [obm-l] RE: [obm-l] Congruência

2011-11-28 Por tôpico Kleber Bastos
13^4=1 mod(10) , elevando o 13 a qualquer potência o 1 poderá se elevar
pela mesma potência (pela proprieda de congruência). Por isso que pulei
direto para  (13)^9^9 = (1)^9^9 mod 10 ...(13)^387420489 = (1)^387420489
mod 10. Ou seja, (13)^9^9 = 1 mod (10)

Não sei se é certo, por isso perguntei.

Abraços, Kleber.

2011/11/28 João Maldonado joao_maldona...@hotmail.com387420489

  Desculpe pelo outro email, saiu errado

 Como você  disse,  13^4 = 1  mod(10), analisando 13, ou 3, mod 10,
  concluimos que 13^n = 1 mod(10) se n = 4k,  ou seja, se n é múltiplo de 4.
   Mas 9^9 não é múltiplo de 4
 Portanto é impossível que 13^(9^9) = 1 mod(10)

 Para  ficar  mais claro

 13^0 = 1  (4k)
 13^1 = 3  (4k+1)
 13^2 = 9  (4k+2)
 13^3 = 7  (4k+3)
 13^4 = 1  (4k)
 13^5 = 3  (4k+1)
 13^6 = 9  (4k+2)
 13^7 = 7  (4k+3)
 13^8 = 1  (4k)
 13^9 = 3  (4k+1)
 13^10 = 9  (4k+2)
 13^11 = 7  (4k+3)
 13^12 = 1  (4k)
 13^13 = 3  (4k+1)
 ...

 O 1, 3, 9, 7  vai se repetindo

 Não entendi essa passagem que você fez

 13^4 = 1 mod(10)
 (13)^9^9 = (1)^9^9 mod(10)


 Sim, 13^4 = 1 mod(10)
 Mas 13 não

 []'s
 Joao

 --
 Date: Mon, 28 Nov 2011 07:27:19 -0200
 Subject: [obm-l] Re: [obm-l] RE: [obm-l] Congruência

 From: klebe...@gmail.com
 To: obm-l@mat.puc-rio.br

 Bom dia, pensei assim:

 13 = 3 mod(10)
 13^2 = -1 mod(10)
 13^4 = -1^2 mod(10)
 13^4 = 1 mod(10)
 (13)^9^9 = (1)^9^9 mod(10)
 (13)^9^9 = 1 mod(10)

 Ou seja, Resto igual a 1. Que será o último algarismo.
 Será que tá errado?

 Abraços, Kleber.

 2011/11/28 João Maldonado joao_maldona...@hotmail.com


 Se for  13^(9^9) mod(10) = 3^(9^9) mod(10)

 Vamos analisar 3^x  mod 10
 3^0 = 1  (4k)
 3^1 = 3  (4k+1)
 3^2 = 9  (4k+2)
 3^3 = 7  (4k+3)

 9^9  mod(4) = 1^9 mod(4) = 1

 Logo  9^9  = 4k+1 e  3^(4k+1 = 3 mod(10)

 Resposta: 3

 --
 Date: Sun, 27 Nov 2011 21:13:15 -0200
 Subject: [obm-l] Congruência
 From: klebe...@gmail.com
 To: obm-l@mat.puc-rio.br


 Olá amigos,

 O exercício é simples, mas não estou conseguindo visualizar essa solução.

 Achar o último algarismo de (13)^9^9 (13 elevado a 9^9).

 Desde de já agradeço a ajuda.

 Abraços,

 --
 Kleber (Ps. fiz por congruência módulo 10, mas não cheguei a conclusão)




 --
 Kleber B. Bastos




-- 
Kleber B. Bastos


[obm-l] RE: [obm-l] Re: [obm-l] RE: [obm-l] Re: [obm-l] RE: [obm-l] Congruência

2011-11-28 Por tôpico João Maldonado


Na verdade é quase isso
13^4 =  1 mod(10),  elevando o 13^4 ( e não o 13) a qualquer potência o 1 será 
elevado à mesma

Date: Mon, 28 Nov 2011 12:51:38 -0200
Subject: [obm-l] Re: [obm-l] RE: [obm-l] Re: [obm-l] RE: [obm-l] Congruência
From: klebe...@gmail.com
To: obm-l@mat.puc-rio.br

13^4=1 mod(10) , elevando o 13 a qualquer potência o 1 poderá se elevar pela 
mesma potência (pela proprieda de congruência). Por isso que pulei direto para  
(13)^9^9 = (1)^9^9 mod 10 ...(13)^387420489 = (1)^387420489 mod 10. Ou seja, 
(13)^9^9 = 1 mod (10)


Não sei se é certo, por isso perguntei.

Abraços, Kleber.

2011/11/28 João Maldonado joao_maldona...@hotmail.com387420489






Desculpe pelo outro email, saiu errado

Como você  disse,  13^4 = 1  mod(10), analisando 13, ou 3, mod 10,  concluimos 
que 13^n = 1 mod(10) se n = 4k,  ou seja, se n é múltiplo de 4.   Mas 9^9 não é 
múltiplo de 4
Portanto é impossível que 13^(9^9) = 1 mod(10)

Para  ficar  mais claro

13^0 = 1  (4k)
13^1 = 3  (4k+1)13^2 = 9  (4k+2)
13^3 = 7  (4k+3)13^4 = 1  (4k)
13^5 = 3  (4k+1)13^6 = 9  (4k+2)
13^7 = 7  (4k+3)13^8 = 1  (4k)
13^9 = 3  (4k+1)13^10 = 9  (4k+2)
13^11 = 7  (4k+3)13^12 = 1  (4k)
13^13 = 3  (4k+1)...

O 1, 3, 9, 7  vai se repetindo

Não entendi essa passagem que você fez

13^4 = 1 mod(10)
(13)^9^9 = (1)^9^9 mod(10)



Sim, 13^4 = 1 mod(10)Mas 13 não

[]'s
Joao
Date: Mon, 28 Nov 2011 07:27:19 -0200
Subject: [obm-l] Re: [obm-l] RE: [obm-l] Congruência
From: klebe...@gmail.com

To: obm-l@mat.puc-rio.br

Bom dia, pensei assim:

13 = 3 mod(10)
13^2 = -1 mod(10)
13^4 = -1^2 mod(10)
13^4 = 1 mod(10)
(13)^9^9 = (1)^9^9 mod(10)

(13)^9^9 = 1 mod(10)

Ou seja, Resto igual a 1. Que será o último algarismo.

Será que tá errado?

Abraços, Kleber.

2011/11/28 João Maldonado joao_maldona...@hotmail.com








Se for  13^(9^9) mod(10) = 3^(9^9) mod(10)
Vamos analisar 3^x  mod 103^0 = 1  (4k)3^1 = 3  (4k+1)3^2 = 9  (4k+2)3^3 = 7  
(4k+3)


9^9  mod(4) = 1^9 mod(4) = 1
Logo  9^9  = 4k+1 e  3^(4k+1 = 3 mod(10)
Resposta: 3

Date: Sun, 27 Nov 2011 21:13:15 -0200
Subject: [obm-l] Congruência


From: klebe...@gmail.com
To: obm-l@mat.puc-rio.br

Olá amigos,

O exercício é simples, mas não estou conseguindo visualizar essa solução.



Achar o último algarismo de (13)^9^9 (13 elevado a 9^9).

Desde de já agradeço a ajuda.

Abraços, 

-- 
Kleber (Ps. fiz por congruência módulo 10, mas não cheguei a conclusão)
  


-- 
Kleber B. Bastos
  


-- 
Kleber B. Bastos
  

[obm-l] Re: [obm-l] RE: [obm-l] Re: [obm-l] RE: [obm-l] Re: [obm-l] RE: [obm-l] Congruência

2011-11-28 Por tôpico Kleber Bastos
Olá João,

Obrigado pelo esclarecimento.

Abração, Kleber.

Em 28 de novembro de 2011 13:06, João Maldonado joao_maldona...@hotmail.com
 escreveu:


 Na verdade é quase isso

 13^4 =  1 mod(10),  elevando o 13^4 ( e não o 13) a qualquer potência o 1
 será elevado à mesma

 --
 Date: Mon, 28 Nov 2011 12:51:38 -0200
 Subject: [obm-l] Re: [obm-l] RE: [obm-l] Re: [obm-l] RE: [obm-l]
 Congruência

 From: klebe...@gmail.com
 To: obm-l@mat.puc-rio.br

 13^4=1 mod(10) , elevando o 13 a qualquer potência o 1 poderá se elevar
 pela mesma potência (pela proprieda de congruência). Por isso que pulei
 direto para  (13)^9^9 = (1)^9^9 mod 10 ...(13)^387420489 = (1)^387420489
 mod 10. Ou seja, (13)^9^9 = 1 mod (10)

 Não sei se é certo, por isso perguntei.

 Abraços, Kleber.

 2011/11/28 João Maldonado joao_maldona...@hotmail.com387420489

  Desculpe pelo outro email, saiu errado

 Como você  disse,  13^4 = 1  mod(10), analisando 13, ou 3, mod 10,
  concluimos que 13^n = 1 mod(10) se n = 4k,  ou seja, se n é múltiplo de 4.
   Mas 9^9 não é múltiplo de 4
 Portanto é impossível que 13^(9^9) = 1 mod(10)

 Para  ficar  mais claro

 13^0 = 1  (4k)
 13^1 = 3  (4k+1)
 13^2 = 9  (4k+2)
 13^3 = 7  (4k+3)
 13^4 = 1  (4k)
 13^5 = 3  (4k+1)
 13^6 = 9  (4k+2)
 13^7 = 7  (4k+3)
 13^8 = 1  (4k)
 13^9 = 3  (4k+1)
 13^10 = 9  (4k+2)
 13^11 = 7  (4k+3)
 13^12 = 1  (4k)
 13^13 = 3  (4k+1)
 ...

 O 1, 3, 9, 7  vai se repetindo

 Não entendi essa passagem que você fez

 13^4 = 1 mod(10)
 (13)^9^9 = (1)^9^9 mod(10)


 Sim, 13^4 = 1 mod(10)
 Mas 13 não

 []'s
 Joao

 --
 Date: Mon, 28 Nov 2011 07:27:19 -0200
 Subject: [obm-l] Re: [obm-l] RE: [obm-l] Congruência

 From: klebe...@gmail.com
 To: obm-l@mat.puc-rio.br

 Bom dia, pensei assim:

 13 = 3 mod(10)
 13^2 = -1 mod(10)
 13^4 = -1^2 mod(10)
 13^4 = 1 mod(10)
 (13)^9^9 = (1)^9^9 mod(10)
 (13)^9^9 = 1 mod(10)

 Ou seja, Resto igual a 1. Que será o último algarismo.
 Será que tá errado?

 Abraços, Kleber.

 2011/11/28 João Maldonado joao_maldona...@hotmail.com


 Se for  13^(9^9) mod(10) = 3^(9^9) mod(10)

 Vamos analisar 3^x  mod 10
 3^0 = 1  (4k)
 3^1 = 3  (4k+1)
 3^2 = 9  (4k+2)
 3^3 = 7  (4k+3)

 9^9  mod(4) = 1^9 mod(4) = 1

 Logo  9^9  = 4k+1 e  3^(4k+1 = 3 mod(10)

 Resposta: 3

 --
 Date: Sun, 27 Nov 2011 21:13:15 -0200
 Subject: [obm-l] Congruência
 From: klebe...@gmail.com
 To: obm-l@mat.puc-rio.br


 Olá amigos,

 O exercício é simples, mas não estou conseguindo visualizar essa solução.

 Achar o último algarismo de (13)^9^9 (13 elevado a 9^9).

 Desde de já agradeço a ajuda.

 Abraços,

 --
 Kleber (Ps. fiz por congruência módulo 10, mas não cheguei a conclusão)




 --
 Kleber B. Bastos




 --
 Kleber B. Bastos




-- 
Kleber B. Bastos